How to prove that the L2 norm is a non-increasing function of time?

In summary, the conversation is about starting a proof for a norm defined by a given equation using the Poincare inequality. The suggester recommends omitting the square root and using the DE to eliminate or modify a certain term. The individual performing the proof follows the suggestion and presents their results. Another individual suggests a different starting point for the proof. The summary ends with the individual asking for confirmation on the sufficiency of their approach.
  • #1
Su3liminal
8
0

Homework Statement


image.jpg



Homework Equations


How can I start the proof? Shall I use the Poincare inequality?

The Attempt at a Solution


Well, I know that this norm is defined by
wmfWa.gif
p4kUw.gif
, but still I don't know how to start constructing the proof?
 
Last edited:
Physics news on Phys.org
  • #2
Su3liminal said:

Homework Statement


image.jpg



Homework Equations


How can I start the proof? Shall I use the Poincare inequality?

The Attempt at a Solution


Well, I know that this norm is defined by
wmfWa.gif
p4kUw.gif
, but still I don't know how to start constructing the proof?

Start by omitting the square root. Take ##\partial / \partial t## inside the integral sign (justify!), then use the DE to eliminate, or at least, modify ##\partial{u^2}/\partial t##.
 
  • Like
Likes 1 person
  • #3
Ray Vickson said:
Start by omitting the square root. Take ##\partial / \partial t## inside the integral sign (justify!), then use the DE to eliminate, or at least, modify ##\partial{u^2}/\partial t##.

Thanks! I have done what you said (note that I just made a change in variables so I stick to the symbol convention of integration by parts.):

[itex]\\2\int_{0}^{L}\frac{\partial^2 s}{\partial x^2}s dx
\\
\\
\\u=s, dv=\frac{\partial^2 s}{\partial x^2}dx
\\du=\frac{\partial s}{\partial x}dx,v=\frac{\partial s}{\partial x}

\\
\\
\\\therefore 2\int_{0}^{L}\frac{\partial^2 s}{\partial x^2}s dx=2s\frac{\partial s}{\partial x}\mid-2\int_{0}^{L}\frac{\partial s}{\partial x}\frac{\partial s}{\partial x}dx=-2\int_{0}^{L}(\frac{\partial s}{\partial x})^{2}dx[/itex]

Is that sufficient?
 
Last edited:
  • #4
Standard for me would be to start with ## \frac{\partial u}{\partial t}= \frac{\partial^2 u}{\partial x^2}## and multiply by ##u## then integrate over the spatial domain ##[0,L]##.
 

Related to How to prove that the L2 norm is a non-increasing function of time?

1. What is the L2 norm?

The L2 norm, also known as the Euclidean norm, is a mathematical concept used to measure the length of a vector in a multi-dimensional space. It is calculated by taking the square root of the sum of squared components of the vector.

2. Why is it important to prove that the L2 norm is a non-increasing function of time?

Proving that the L2 norm is a non-increasing function of time is important in many fields, such as physics, engineering, and data analysis. It can help in understanding the behavior of systems over time and making predictions about their future states.

3. How can I prove that the L2 norm is a non-increasing function of time?

The most common way to prove that the L2 norm is a non-increasing function of time is by using mathematical techniques, such as calculus and linear algebra. This involves manipulating the equations that define the L2 norm and showing that it decreases over time.

4. What are the implications of the L2 norm being a non-increasing function of time?

If the L2 norm is proven to be a non-increasing function of time, it means that the vector being measured is becoming more stable or localized over time. This could indicate a steady state or convergence towards a specific value.

5. Are there any limitations to proving that the L2 norm is a non-increasing function of time?

While proving the L2 norm to be a non-increasing function of time is useful in many cases, it may not always be applicable. In some situations, other norms or measures may be more appropriate for capturing the behavior of a system over time.

Similar threads

  • Calculus and Beyond Homework Help
Replies
1
Views
579
  • Calculus and Beyond Homework Help
Replies
1
Views
1K
  • Calculus and Beyond Homework Help
Replies
5
Views
2K
  • Calculus and Beyond Homework Help
Replies
12
Views
2K
  • Calculus and Beyond Homework Help
Replies
5
Views
3K
  • Calculus and Beyond Homework Help
Replies
1
Views
915
  • Calculus and Beyond Homework Help
Replies
5
Views
1K
  • Calculus and Beyond Homework Help
Replies
9
Views
2K
  • Calculus and Beyond Homework Help
Replies
5
Views
1K
  • Calculus and Beyond Homework Help
Replies
9
Views
2K
Back
Top